Robins RBCs

Ace your homework & exams now with Quizwiz!

D The acute blood loss, in this case intraperitoneal hemor- rhage, results in a reticulocytosis from marrow stimulation by anemia. Basophilic stippling of RBCs suggests a marrow injury, such as with a drug or toxin. Hypochromic RBCs occur in iron deficiency and thalassemias, both associated with reduced he- moglobin synthesis. Acute blood loss does not give rise to iron deficiency if iron stores and diet are adequate. Leukoerythro- blastosis is typical of a myelophthisic process in the marrow, with both immature WBCs (myelocytes) and RBCs (nucleated forms) present. Schistocytes suggest a microangiopathic hemo- lytic anemia, which can accompany shock or sepsis

2 A healthy 19-year-old woman suffered blunt abdominal trauma in a motor vehicle accident. On admission to the hos- pital, her initial hematocrit was 33%, but over the next hour, it decreased to 28%. A paracentesis yielded serosanguineous fluid. She was taken to surgery, where a liver laceration was repaired, and 1 L of bloody fluid was removed from the peri- toneal cavity. She remained stable. A CBC performed 3 days later is most likely to show which of the following morpho- logic findings in the peripheral blood? A Basophilic stippling of red cells B Hypochromic red cells C Leukoerythroblastosis D Reticulocytosis E Schistocytosis

62 D Because platelets must be stored at room tempera- ture, any contamination that occurred at the time of collec- tion, such as bacterial skin flora of the donor, is more likely to cause sepsis in the recipient. The other products listed are refrigerated, which reduces bacterial growth. The causative agent of syphilis, Treponema pallidum, is often killed at refrig- erator temperatures.

62 A study of transfusion reactions reveals that some patients experienced an increase in body temperature that was greater than 1°C, accompanied by chills and hypoten- sion. Blood cultures are positive for bacterial organisms. These patients respond to antibiotic therapy. Which of the following types of blood products did they most likely receive? A CryoprecipitateB Fresh frozen plasma C GranulocytesD PlateletsE Red blood cells

D In sickle cell anemia, the cumulative ischemic dam- age to the spleen results in autosplenectomy, leaving behind a small fibrotic remnant of this organ. The impaired splenic function and resultant inability to clear bacteria from the bloodstream can occur early in childhood, leading to risk for infection with encapsulated bacterial organisms. Immunode- ficiency results from lack of splenic function, not from lack of immunoglobulins. Endothelium can be damaged with sickling, and adhesion between endothelial cells and RBCs is increased in sickle cell anemia. Complement proteins are part of innate immune responses in acute inflammation. There is no impairment in production or function of neutrophils.

A 10-year-old child has experienced multiple episodes of pneumonia and meningitis with septicemia since infancy. Causative organisms include Streptococcus pneumoniae and Haemophilus influenzae. On physical examination, the child has no organomegaly and no deformities. Laboratory studies show hemoglobin of 9.2 g/dL, hematocrit of 27.8%, platelet count of 372,000/mm3, and WBC count of 10,300/mm3. A he- moglobin electrophoresis shows 1% hemoglobin A2, 7% he- moglobin F, and 92% hemoglobin S. Which of the following is the most likely cause of the repeated infections in this child? A Absent endothelial cell expression of adhesion molecules B Diminished hepatic synthesis of complement proteins C Impaired neutrophil production D Loss of normal splenic function E Reduced synthesis of immunoglobulins

E Children and adults with sickle cell anemia may ben- efit from hydroxyurea therapy, which can increase the con- centration of hemoglobin F in RBCs, which interferes with the polymerization of hemoglobin S. However, the therapeutic response to hydroxyurea often precedes the increase in hemo- globin F levels. Hydroxyurea also has an anti-inflammatory effect, increases the mean RBC volume, and can be oxidized by heme groups to produce nitric oxide that promotes vaso- dilation. Because hemoglobin F levels remain high through the first 5 to 6 months of life, patients with sickle cell ane- mia typically do not manifest the disease during infancy. Because both β-globin chains are affected, no hemoglobin A1 is produced, and A2 levels are never high. Globin synthesis overall is not going to increase, and globin synthesis must be balanced to produce normal hemoglobin. The hemolysis associated with sickling promotes erythropoiesis, but the concentration of hemoglobin S is not changed. Hydroxyurea does not significantly shift the oxygen dissociation curve or change the oxygen affinity of the various hemoglobins.

A 12-year-old boy has a history of episodes of severe ab- dominal, chest, and back pain since early childhood. On physi- cal examination, he is afebrile, and there is no organomegaly. Laboratory studies show hemoglobin of 11.2 g/dL, platelet count of 194,000/mm3, and WBC count of 9020/mm3. The pe- ripheral blood smear shows occasional sickled cells, nucleated RBCs, and Howell-Jolly bodies. Hemoglobin electrophoresis shows 1% hemoglobin A2, 6% hemoglobin F, and 93% hemo- globin S. Hydroxyurea therapy is found to be beneficial in this patient. An increase in which of the following is the most likely basis for its therapeutic efficacy? A Erythrocyte production B Overall globin chain synthesis C Oxygen affinity of hemoglobin D Production of hemoglobin A E Production of hemoglobin F

56 E His hemophilia A is monitored with factor VIII activ- ity. Hemophilia B with factor IX deficiency would present sim- ilar findings, but it is much less common. Transfusion of factor VIII helps to prevent joint and soft-tissue hemorrhages. Of in- dividuals with hemophilia A, 20% may develop an inhibitor to factor VIII, typically an IgG antibody that neutralizes activ- ity of any infused factor VIII. In this case, the partial thrombo- plastin time partly corrects, indicating that some inhibitor may be present. Mucocutaneous bleeding is more typical for plate- let disorders and von Willebrand disease. With hemophilia, bleeding is often into joints and soft tissues. The prothrombin time (PT) is normal, and clotting factors affecting this pathway are made in the liver; factor VIII is in the intrinsic pathway measured by partial thromboplastin time (PTT). Hemophilia A and hemophilia B are X-linked inherited conditions, so males are mainly affected. There can be rare new mutations (such as in Queen Victoria), which introduce the gene into a family.

A 12-year-old boy has had worsening problems with joint mobility involving his arms and legs, particularly his knees and ankles, for the past 6 years. He has been receiving therapy for this condition. His grandfather had a similar condition and died at age 25 years. On physical examination, he has no visible pe- techiae or areas of purpura. Laboratory studies show that pro- thrombin time is 12 seconds, and partial thromboplastin time is 52 seconds. After addition of an equivalent aliquot of normal plasma, the partial thromboplastin time is 30 seconds. Hemo- globin is 12.9 g/dL, platelet count is 238,500/mm3, and WBC count is 6620/mm3. His platelet function studies are normal. What is the most likely inheritance pattern for his condition? A Autosomal dominant B Autosomal recessive C Confined placental mosaicism D Germline mutation E X-linked recessive

C The crescent-shaped RBCs (sickled RBCs) are char- acteristic of hemoglobin SS. This disease is most common in individuals of African and eastern Arabian descent. The sick- led RBCs are susceptible to hemolysis (mainly vascular, in the spleen), but they also can cause microvascular occlusions anywhere in the body, most commonly bone, lungs, liver, and brain, leading to ischemia and severe pain. Vascular occlusions in the lungs are often accompanied by infection and lead to "acute chest syndrome." Abdominal pain and back pain are common and severe, requiring prompt and effective analgesia. The cell membranes of reversibly sickled cells are abnormally "sticky," and they adhere to capillary endothelium, especially in lungs. Vasoconstriction is caused by depletion of NO by free hemoglobin. Adhesion of RBCs to endothelium retards blood flow, creates hypoxia, and precipitates local sickling and vas- cular occlusion. Chronic tissue hypoxia does occur in sickle cell anemia, but it produces insidious impairment of function in organs such as heart, kidneys, and lungs. Defects in the alter- native pathway of complement activation predispose to infec- tion with encapsulated bacteria, such as Haemophilus influenzae and Streptococcus pneumoniae. Autoantibodies to alveolar base- ment membrane can be part of Goodpasture syndrome, which also affects kidneys. The most severe intravascular hemolysis occurs with major transfusion reactions.

A 13-year-old boy has the sudden onset of severe ab- dominal pain and cramping accompanied by chest pain, non- productive cough, and fever. On physical examination, his temperature is 39° C, pulse is 110/min, respirations are 22/min, and blood pressure is 80/50 mm Hg. He has diffuse abdominal tenderness, but no masses or organomegaly. Laboratory studies show a hematocrit of 18%. The peripheral blood smear is shown in the figure. A chest radiograph shows bilateral pulmonary in- filtrates. Which of the following is the most likely mechanism for initiation of his pulmonary problems? A Chronic hypoxia of the pulmonary parenchyma B Defects in the alternative pathway of complement activation C Extensive RBC adhesion to endothelium D Formation of autoantibodies to alveolar basement membrane E Intravascular antibody-induced hemolysis

58 B The history in this case is similar to that in the preced- ing question; however, the partial thromboplastin time (PTT) is corrected by normal pooled plasma. The patient has hemo- philia B caused by factor IX deficiency, and inhibitors of factor IX are absent from the patient's serum. How is this possible in a female patient? X-inactivation ("unfavorable lyonization") can explain this phenomenon and could explain why female carriers of hemophilia A or B have a tendency to bleed. ("When you have eliminated the impossible, that which remains, how- ever improbable, must be the truth," said Sherlock Holmes in The Sign of Four.) An in vitro mixing study of patient and pooled plasma such as this usually corrects an abnormality caused by a deficiency of a procoagulant factor, but if there is a coagulation inhibitor in the patient's plasma, the clotting test would show an abnormal result. The mixing study excludes the antiphospholipid syndrome. Idiopathic thrombocytopenic purpura is characterized by the presence of antiplatelet anti- bodies and thrombocytopenia. Thrombotic thrombocytopenic purpura is a microangiopathic hemolytic anemia characterized by renal failure and central nervous system abnormalities. Von Willebrand disease is caused by decreased platelet adhe- sion and has features resembling thrombocytopenia

A 15-year-old girl has a history of easy bruising and hemorrhages. Since menarche at the age of 13 years, she has had menometrorrhagia. On physical examination, she dis- plays joint deformity and has decreased mobility of the an- kles, knees, and wrists. Laboratory studies show hemoglobin, 11.8 g/dL; hematocrit, 35.1%; platelet count, 267,000/mm3; WBC count, 5960/mm3; prothrombin time, 13 seconds; and partial thromboplastin time, 60 seconds. A 1:1 dilution of the patient's plasma with normal pooled plasma corrects the par- tial thromboplastin time. Which of the following is the most likely diagnosis? A Antiphospholipid syndrome B Hemophilia B C Idiopathic thrombocytopenic purpura D Thrombotic thrombocytopenic purpura E Von Willebrand disease

D Paroxysmal nocturnal hemoglobinuria (PNH) is a disorder that results from an acquired stem cell membrane defect produced by a PIGA gene mutation that prevents the membrane expression of certain proteins that require a gly- colipid anchor. These include proteins that protect cells from lysis by spontaneously activated complement. As a result, RBCs, granulocytes, and platelets are exquisitely sensitive to the lytic activity of complement. The RBC lysis is intra- vascular, so patients can have hemoglobinuria (dark urine). Defects in platelet function are believed to be responsible for venous thrombosis. Recurrent infections can be caused by impaired leukocyte functions. Patients with PNH may develop acute leukemia or aplastic anemia as complications. Mutations in the β-globin chain can give rise to hemoglo- binopathies such as sickle cell anemia. Patients with factor V (Leiden) and prothrombin G20210A mutations can pres- ent with thromboses, but there is no anemia or leukopenia. Patients with glucose-6-phosphate dehydrogenase (G6PD) deficiency have an episodic course from exposure to agents such as drugs that induce hemolysis. Spectrin mutations give rise to hereditary spherocytosis.

A 16-year-old boy notes passage of dark urine. He has a history of multiple bacterial infections and venous thromboses for the past 10 years, including portal vein thrombosis in the previous year. On physical examination, his right leg is swol- len and tender. CBC shows hemoglobin, 9.8 g/dL; hematocrit, 29.9%; MCV, 92 μm3; platelet count, 150,000/mm3; and WBC count, 3800/mm3 with 24% segmented neutrophils, 1% bands, 64% lymphocytes, 10% monocytes, and 1% eosinophils. He has a reticulocytosis, and his serum haptoglobin level is very low. A mutation affecting which of the following gene prod- ucts is most likely to give rise to this clinical condition? A β-Globin chain B Factor V C Glucose-6-phosphate dehydrogenase D Phosphatidylinositol glycan A (PIGA) E Prothrombin G20210A F Spectrin

59 B Glanzmann thrombasthenia is a rare autosomal re- cessive disorder with defective platelet aggregation from de- ficiency or dysfunction of glycoprotein IIb/IIIa. The platelet aggregation studies described here are characteristic for this disorder. Disseminated intravascular coagulation results in consumption of all coagulation factors and platelets, so the prothrombin time and partial thromboplastin time are el- evated with thrombocytopenia. Immune thrombocytopenic purpura is caused by antibodies to platelet membrane gly- coproteins IIb/IIIa or Ib/IX. Scurvy resulting from vitamin C deficiency causes bleeding into soft tissues and skin from increased capillary fragility, but platelet number and func- tion are normal. Von Willebrand disease is one of the most common bleeding disorders and results from qualitative or quantitative defects in von Willebrand factor.

A 16-year-old girl has had frequent nosebleeds since childhood. Her gums bleed easily, even with routine tooth brushing. She has experienced menorrhagia since menarche at age 13 years. On physical examination, there are no ab- normal findings. Laboratory studies show hemoglobin, 14.1 g/dL; hematocrit, 42.5%; MCV, 90 μm3; platelet count, 277,400/mm3; and WBC count, 5920/mm3. Her platelets fail to aggregate in response to ADP, collagen, epinephrine, and thrombin. The ristocetin agglutination test result is normal. There is a deficiency of glycoprotein IIb/IIIa. Prothrombin time is 12 seconds, and partial thromboplastin time is 28 seconds. What is the most likely diagnosis? A Disseminated intravascular coagulation B Glanzmann thrombasthenia C Immune thrombocytopenic purpura D Vitamin C deficiency E Von Willebrand disease

B This patient has β-thalassemia, probably of at least intermediate severity. There is decreased β-globin chain for- mation, with increased hemoglobin A2 and F to compensate. There is ineffective erythropoiesis and increased erythropoie- tin to drive increased iron absorption, leading to iron overload. Chronic anemia requiring transfusion therapy exacerbates he- mochromatosis. Iron deposited in endocrine tissues can lead to gonadal, pituitary, thyroid, islet cell, and adrenal failure. Secondary hypersplenism can result from the splenomegaly, with sequestration of platelets and leukocytes. The abnormal ankyrin gene leads to hereditary spherocytosis and a mild he- molytic anemia with splenomegaly, but not to iron overload. In glucose-6-phosphate dehydrogenase deficiency, sensitivity to oxidizing agents causes a hemolytic anemia, but this usually is not ongoing. The HFE gene is abnormal in hereditary hemo- chromatosis, leading to iron overload, but onset of the disease occurs in middle age. Mutations involving NADPH oxidase lead to immunodeficiency in chronic granulomatous disease.

A 17-year-old girl has had a history of fatigue and weak- ness for her entire life. She has not undergone puberty. On physical examination, secondary sex characteristics are not well developed. She has hepatosplenomegaly. CBC shows he- moglobin of 9.1 g/dL, hematocrit of 26.7%, MCV of 66 μm3, platelet count of 89,000/mm3, and WBC count of 3670/mm3. The appearance of the peripheral blood smear is shown in the figure. Additional laboratory findings include serum glucose of 144 mg/dL, TSH of 6.2 mU/mL, and ferritin of 679 ng/mL. A mutation in a gene encoding for which of the following is most likely to be present in this girl? A Ankyrin B β-Globin C G6PD D HFE E NADPH oxidase

49 B Heparin-induced thrombocytopenia affects 3% to 5% of patients treated for 1 to 2 weeks with unfractionated heparin. These patients form IgG antibodies to heparin-platelet factor 4 complexes that bind to Fc receptors on the surface of platelets, causing platelet activation and, paradoxically, thrombosis. Aspirin has antiplatelet effects that take days to occur, and bleeding (not thrombosis) is the major risk. Tissue plasminogen activator and urokinase are fibrinolytic agents, with the former used acutely to treat conditions such as coronary thrombosis, although the latter also may be used for venous clot lysis. Warfarin (Coumadin) was avoided in this patient because of the protein C deficiency; typically, the patient is switched from heparin to warfarin. Warfarin therapy prolongs the prothrombin time by interfering with vitamin K-dependent clotting factor synthesis in the liver.

A 21-year-old woman known to have a protein C defi- ciency develops recurrent pulmonary thromboembolism and is placed on anticoagulant therapy. Two weeks after initiation of this therapy, she has a sudden change in mental status and experiences difficulty speaking and swallowing. A cerebral angiogram shows a left middle cerebral artery occlusion. Lab- oratory studies show hemoglobin of 13 g/dL, platelet count of 65,400/mm3, WBC count of 5924/mm3, prothrombin time of 12 seconds, and partial thromboplastin time of 51 seconds. The anticoagulant therapy is discontinued. Which of the following pharmacologic agents used as an anticoagulant in this patient is most likely to have caused these findings? A Acetylsalicylic acid (aspirin) B Heparin C Tissue plasminogen activator D Urokinase E Warfarin

E This is benign tertian malaria. The bite of the Anophe- les mosquito introduces sporozoites, which travel to the liver to reproduce. The resulting merozoites are released into the bloodstream and infect RBCs. Asexual reproduction within the RBCs yields trophozoites, and periodic hemolysis with release of the parasites produces the characteristic clinical findings. Aspergillus organisms invade blood vessels and cause thrombosis, but hemolysis of RBCs is inconsequential. Babesiosis is far less common than malaria, is endemic to the northeastern United States, and does not produce episodic fevers. Dirofilaria is the heartworm found in dogs, which rarely infects humans and does not cause hemolysis. Similar to other gram-negative bacteria, Escherichia coli can release lipopolysaccharide, which causes severe sepsis and possible disseminated intravascular coagulation, a microangiopathic hemolytic anemia. Wuchereria bancrofti is a nematode that prefers to live in lymphatics.

A 22-year-old woman after returning from a trip to Africa has experienced febrile episodes over the past 2 weeks. On physical examination, her temperature is 37.5° C, pulse is 82/min, respirations are 18/min, and blood pressure is 105/65 mm Hg. Laboratory studies show hemoglobin of 10.8 g/dL, hematocrit of 32.5%, platelet count of 245,700/mm3, and WBC count of 8320/mm3. The serum haptoglobin level is decreased, and direct and indirect Coombs test results are negative. The reticulocyte count is increased. The prothrombin time is 12 seconds, and the partial thromboplastin time is 31 seconds. She is observed over the next week and found to have tem- perature spikes to 39.1° C, with shaking chills every 48 hours. Infection with which of the following organisms is most likely to cause this patient's illness? A Aspergillus niger B Babesia microti C Dirofilaria immitis D Escherichia coli E Plasmodium vivax F Wuchereria bancrofti

B Cold agglutinin disease has antibody (usually IgM) coating RBCs. The IgM antibodies bind to the RBCs at low temperature at peripheral body sites and fix complement; however, complement is not lytic at this temperature. With an increase in temperature within core internal organs, the IgM is dissociated from the cell, leaving behind C3b. Most of the hemolysis occurs extravascularly in the cells of the mononuclear phagocyte system, such as Kupffer cells in the liver, or splenic macrophages, because the coating of complement C3b acts as an opsonin. IgG is typically involved in warm antibody hemolytic anemia, which is chronic and is not triggered by cold. Raynaud phenom- enon occurs in exposed, colder areas of the body, such as the fingers and toes. The patient probably has an elevated cold agglutinin titer. Histamine is released in type I hyper- sensitivity reactions. Fibronectin is an adhesive cell sur- face glycoprotein that aids in tissue healing. IgE is present in allergic conditions.

A 22-year-old woman has experienced malaise and a sore throat for 2 weeks. Her fingers turn white on exposure to cold. On physical examination, she has a temperature of 37.8° C, and the pharynx is erythematous. Laboratory find- ings include a positive monospot (heterophile antibody) test result. Direct and indirect Coombs test results are positive at 4° C, although not at 37° C. Which of the following molecules bound on the surfaces of the RBCs most likely accounts for these findings? A α2-Macroglobulin B Complement C3b C Fibronectin D Histamine E IgE

C Glucose-6-phosphate dehydrogenase (G6PD) defici- ency is an X-linked disorder that affects about 10% of African- American males. The lack of this enzyme subjects hemoglobin to damage by oxidants, including drugs such as primaquine, sulfonamides, nitrofurantoin, phenacetin, and aspirin (in large doses). Infection can also cause oxidative damage to he- moglobin. Heinz bodies are denatured hemoglobin, and they damage the RBC membrane, giving rise to intravascular he- molysis. The "bite cells" result from the attempts of overeager splenic macrophages to pluck out the Heinz bodies, adding an element of extravascular hemolysis. Heterozygotes with α-thalassemia (1 or 2 abnormal genes out of 4 total α-globin genes) have no major problems, but in cases of α-thalassemia major, perinatal death is the rule. Likewise, β-thalassemia minor and sickle cell trait are conditions usually with no ma- jor problems and no relation to drug usage. RBC membrane abnormalities, such as hereditary spherocytosis (caused by abnormal spectrin), typically produce a mild anemia without significant hemolysis, and there is no drug sensitivity. Some autoimmune hemolytic anemias can be drug related, but the hemolysis is predominantly extravascular.

A 23-year-old African-American man passes dark red- dish brown urine 3 days after taking an anti-inflammatory medication that includes phenacetin. He is surprised, because he has been healthy all his life and has had no major illnesses. On physical examination, he is afebrile, and there are no re- markable findings. CBC shows a mild normocytic anemia, but the peripheral blood smear shows precipitates of denatured globin (Heinz bodies) with supravital staining and scattered "bite cells" in the population of RBCs. Which of the following is the most likely diagnosis? A α-Thalassemia minor B β-Thalassemia minor C Glucose-6-phosphate dehydrogenase deficiency D Sickle cell trait E Abnormal ankyrin in RBC cytoskeletal membrane F Warm antibody autoimmune hemolytic anemia

52 B An inherited bleeding disorder with normal platelet count and prolonged bleeding time suggests von Willebrand disease, confirmed by the ristocetin-dependent bioassay for von Willebrand factor (vWF). Von Willebrand disease is a common bleeding disorder, with an estimated frequency of 1%. In most cases, it is inherited as an autosomal domi- nant trait. In these cases, a reduction in the quantity of vWF impairs platelet adhesion to damaged vessel walls, and he- mostasis is compromised. Because vWF acts as a carrier for factor VIII, the level of this procoagulant protein (needed for the intrinsic pathway) is diminished, as in this case. The lev- els of factor VIII rarely are reduced enough, however, to be clinically significant, perhaps 1 in 10,000 persons. Prolonged partial thromboplastin time corrected by normal plasma is a reflection of factor VIII deficiency. Because the disease is not a disorder of stem cells, bone marrow failure or myeloprolif- erative disorder is not a likely outcome. Joint hemorrhages are a feature of hemophilia A and B, not von Willebrand disease. Patients with von Willebrand disease are not prone to thrombosis, as are individuals with factor V (Leiden) mu- tation or other inherited disorders of anticoagulation

A 23-year-old woman has a history of easy bruising. Physical examination shows multiple bruises ranging in color from red to blue to purple on her arms and legs. There is no or- ganomegaly, and no deformities are noted. Laboratory studies show hemoglobin, 9.5 g/dL; hematocrit, 28.2%; platelet count, 229,300/mm3; WBC count, 7185/mm3; prothrombin time, 12 seconds; and partial thromboplastin time, 38 seconds. A 1:1 dilution of the patient's plasma with normal pooled plasma corrects the partial thromboplastin time. Ristocetin-dependent platelet aggregation in patient plasma is markedly reduced. Factor VIII activity is 30% (reference range 50% to 150%) Which of the following is the most likely potential consequence of this disease? A Bone marrow failure from aplasia B Excessive bleeding after oral surgery C Increasing difficulty with joint mobility D Myeloproliferative disorder E Recurrent deep venous thrombosis

45 A The presence of thrombocytopenia, increased pro- thrombin and partial thromboplastin times, and fibrin split products, and the low fibrinogen concentration all suggest disseminated intravascular coagulation (DIC), which was most likely caused by a retained dead fetus. This obstetric complication can lead to DIC through release of thrombo- plastins from the fetus. The thromboplastins cause wide- spread microvascular thrombosis and consume clotting factors and platelets. There is no defect in platelet function. There is no damage to the vascular endothelium or vascu- lar wall. Platelet production is normal, but platelets are con- sumed by widespread thrombosis of small vessels.

A 23-year-old woman in her 25th week of pregnancy has felt no fetal movement for the past 3 days. Three weeks later, she still has not given birth and suddenly develops dyspnea with cyanosis. On physical examination, her temperature is 37° C, pulse is 106/min, respirations are 23/min, and blood pres- sure is 80/40 mm Hg. She has large ecchymoses over the skin of her entire body. A stool sample is positive for occult blood. Laboratory studies show an elevated prothrombin time and partial thromboplastin time. The platelet count is decreased, plasma fibrinogen is markedly decreased, and fibrin split prod- ucts are detected. A blood culture is negative. Which of the fol- lowing is the most likely cause of her bleeding diathesis? A Consumption of coagulation factors B Defects in platelet aggregation C Increased vascular fragility D Reduced production of platelets E Toxic injury to the endothelium

60 D The findings are those of a hemolytic (ABO) trans- fusion reaction, virtually all of which result from clerical errors. You learned that proper phlebotomy procedure re- quires labeling the tubes just after drawing the blood, not handing the tubes to someone else for labeling and possible mix-up. This is not consistent with a transfusion-related acute lung injury (TRALI) in which there are donor-derived HLA or granulocyte-specific antibodies in the recipient's blood product that are directed against antigens on the reci- pient WBCs. Though hepatitis C infection is still possible, but uncommon, from transfused blood, this infection has an incubation period of months. The two units of packed RBCs in a young person are very unlikely to lead to circu- latory overload. He does not have transfusion-associated graft-versus-host disease, which is a rare but untreatable condition with a very high mortality rate.

A 25-year-old man involved in a motorcycle accident incurs a laceration to his thigh. The bleeding is stabilized en route to the hospital, but on arrival he is noted to have or- thostatic hypotension and his hematocrit is 21%. He receives 2 units of PRBCs. As the first unit is nearly finished transfus- ing, he becomes febrile and hypotensive. Urine output ceases. The serum above the clot in a red top phlebotomy tube is pink. Which of the following complications of transfusion has most likely occurred in this man? A Donor antibodies were directed against his granulocytes B Donor blood was contaminated with hepatitis C virus C Fluid overload led to congestive heart failure D Mislabeled specimens were processed in the laboratory E Foreign T lymphocytes attacked his tissues

C Although β-thalassemia minor and iron deficiency anemia are both characterized by hypochromic and micro- cytic RBCs, there is no increase in hemoglobin A2 in iron de- ficiency states. A normal serum ferritin level also excludes iron deficiency. In contrast to β-thalassemia major, there is usually a mild anemia without major organ dysfunction with β-thalassemia minor. Diseases that produce hemolysis and increase erythropoiesis (e.g., autoimmune hemolytic anemia, malaria) do not alter the composition of β-globin chain pro- duction. Anemia of chronic disease may mimic iron deficien- cy and thalassemia minor with respect to hypochromia and microcytosis; however, anemia of chronic disease is associ- ated with an increase in the serum concentration of ferritin

A 25-year-old woman has a 3-year history of arthralgias. Physical examination shows no joint deformity, but she ap- pears pale. Laboratory studies show total RBC count of 4.7 million/mm3, hemoglobin of 12.5 g/dL, hematocrit of 37.1%, platelet count of 217,000/mm3, and WBC count of 5890/mm3. The peripheral blood smear shows hypochromic and micro- cytic RBCs. Total serum iron and ferritin levels are normal. Hemoglobin electrophoresis shows 93% hemoglobin A1 with elevated hemoglobin A2 level of 5.8% and hemoglobin F level of 1.2%. What is the most likely diagnosis? A Anemia of chronic disease B Autoimmune hemolytic anemia C β-Thalassemia minor D Infection with Plasmodium vivax E Iron deficiency anemia

F Hereditary spherocytosis is a condition in which a mutation affects one of several membrane cytoskeletal pro- teins. Spectrin and related proteins are cytoskeletal proteins that are important in maintaining the RBC shape. These pro- teins include ankyrin (most common) and band 4.2, which binds spectrin to the transmembrane ion transporter; band 3; and protein 4.1, which binds the "tail" of spectrin to another transmembrane protein, glycophorin A. The abnormal RBCs with such mutant proteins are less deformable, lack central pallor on a peripheral blood smear, and they are sequestered and destroyed in the spleen. Thalassemias with abnormal α-globin or β-globin chains are associated with hypochromic microcytic anemias. Iron deficiency affects the heme portion of hemoglobin, leading to hypochromia and to microcyto- sis. Carbonic anhydrase in RBCs helps to maintain buffering capacity. Glucose-6-phosphate dehydrogenase (G6PD) defi- ciency is an X-linked condition that most commonly affects black males. Porphyrias may affect the production of por- phyrin rings and may lead to hemolytic anemia along with abdominal pain, neurologic problems, or skin findings

A 28-year-old woman has had a constant feeling of leth- argy since childhood. On physical examination, she is afebrile and has a pulse of 80/min, respirations of 15/min, and blood pressure of 110/70 mm Hg. The spleen tip is palpable, but there is no abdominal pain or tenderness. Laboratory stud- ies show hemoglobin of 11.7 g/dL, platelet count of 159,000/ mm3, and WBC count of 5390/mm3. The peripheral blood smear shows small round erythrocytes that lack a zone of cen- tral pallor. An inherited abnormality in which of the following RBC components best accounts for these findings? A α-Globin chain B β-Globin chain C Carbonic anhydrase D Glucose-6-phosphate dehydrogenase E Heme with porphyrin ring F Spectrin cytoskeletal protein

38 A Aplastic anemia leads to marked pancytopenia. Many cases are idiopathic, although some can follow toxic exposures to chemotherapy drugs or to chemicals, such as benzene. Some cases may follow viral hepatitis infections. An intrinsic defect in stem cells, or T lymphocyte suppres- sion of stem cells, can play a role in the development of aplastic anemia. Hemolysis is unlikely because the bilirubin is normal, and there is no history of an autoimmune disease. An increased susceptibility to complement lysis occurs in paroxysmal nocturnal hemoglobinuria as a result of muta- tions in the PIGA gene. It is unlikely that the patient has met- astatic disease at this age, with no prior illness; metastases are more likely to produce a leukoerythroblastic peripheral blood appearance. Sequestration of peripheral blood cells in an enlarged spleen could account for mild pancytopenia, but in this case the spleen is not enlarged.

A 28-year-old, previously healthy man has noted increasing fatigue for the past 6 months and formation of bruises after minimal trauma. Over the past 2 days, he has developed a cough. On physical examination, his temperature is 38.9° C, and he has diffuse rales in both lungs. He has no hepatospleno- megaly and no lymphadenopathy. Laboratory findings in- clude a sputum culture positive for Streptococcus pneumoniae, hemoglobin of 7.2 g/dL, hematocrit of 21.7%, platelet count of 23,400/mm3, WBC count of 1310/mm3, prothrombin time of 13 seconds, partial thromboplastin time of 28 seconds, and total bilirubin of 1 mg/dL. The ANA test result is negative. What is the most likely explanation of these findings? A Hematopoietic stem cell defect B Hemolysis of antibody-coated cells C Increased susceptibility to lysis by complement D Metastatic adenocarcinoma to bone marrow E Secondary hypersplenism

B Mechanical trauma to RBCs is possible, but typi- cally is not severe. It can follow strenuous exercises involv- ing repeated blows to body parts. Complement-mediated lysis is a feature of immunohemolytic anemias. Increased osmotic fragility is noted in spherocytes. Sickle cell anemia is not likely to be found in the population groups in the coun- tries noted, and persons with this disease are not likely to be playing rugby. Splenic sequestration is a feature of hemolytic anemias due to membrane defects and antibodies.

A 29-year-old rugby player takes part in a particularly con- tentious game between New Zealand and South Africa. He is the forward prop in the scrums, hitting hard and being hit hard by other players. He feels better after downing several pints of beer following the game, but notes darker urine. Urinalysis is posi- tive for blood. Which of the following pathogenic mechanisms underlies change in the color of urine? A Complement lysis B Intravascular disruption C Osmotic fragility D Sinusoidal sickling E Splenic sequestration

C She has a circulating antibody against her RBCs lead- ing to hemolytic anemia. The indirect antiglobulin (Coombs) test detects antibody in the plasma. The direct antiglobulin (Coombs) test detects antibody bound to RBCs. Autoimmune hemolytic anemias can be a feature of autoimmune diseases, such as systemic lupus erythematosus in this woman. Most of the hemolysis is extravascular in the spleen, but some can be intravascular. The reticulocyte count is typically increased (polychromasia) with hemolysis, and serum haptoglobin is diminished. An elevated D-dimer suggests a microangio- pathic hemolytic anemia, but she has no schistocytes on the peripheral blood smear. Increased osmotic fragility is a fea- ture of RBCs in paroxysmal nocturnal hemoglobinuria. Her mild macrocytosis indicates increased reticulocytosis, not vi- tamin B12 deficiency, and hemolysis is not part of pernicious anemia. Increased serum ferritin is typical for anemia of chronic disease (mild increase) or hemochromatosis (marked increase).

A 29-year-old woman has had fatigue with dizziness for the past 5 months. On physical examination, she has an ery- thematous malar rash. She has no lymphadenopathy, but there is a palpable spleen tip. She is afebrile. Laboratory studies show hemoglobin, 8.9 g/dL; hematocrit, 27.8%; MCV, 103 μm3; RBC distribution width index, 22; WBC count, 8650/mm3; platelet count, 222,000/mm3; and reticulocyte count, 3.3%. The periph- eral blood smear shows polychromasia, but no schistocytes. Her serum total bilirubin is 3.2 mg/dL with direct bilirubin 0.8 mg/dL, and haptoglobin is 5 mg/dL. Antinuclear antibody and anti-double-stranded DNA tests are positive. What addi- tional laboratory test finding is she most likely to have? A D-dimer 10 μg/mL B Increased RBC osmotic fragility C Positive Coombs test D Serum cobalamin (vitamin B12) 50 pg/mL E Serum ferritin 240 ng/mL

A This patient of Mediterranean descent has β-thalassemia major. In this condition, there is a severe reduction in the synthesis of β-globin chains without impairment of α-globin synthesis. The free, unpaired α-globin chains form aggre- gates that precipitate within normoblasts and cause them to undergo apoptosis. The death of RBC precursors in the bone marrow is called "ineffective erythropoiesis." Not only does this cause anemia, but it also increases the absorption of dietary iron, giving rise to iron overload, which results in hemochro- matosis with infiltrative cardiomyopathy, hepatic cirrhosis, and "bronze diabetes" from pancreatic islet dysfunction. The severe anemia triggers erythropoietin synthesis, which expands the erythropoietic marrow. The marrow expansion encroaches on the bones, causing maxillofacial deformities. Extramedullary hematopoiesis causes hepatosplenomegaly. In comparison, the hemolytic anemia is mild in β-thalassemia minor, and there is very little ineffective erythropoiesis. He- mochromatosis is particularly detrimental to the liver and heart. Patients with chronic anemia may require RBC trans- fusions, which adds even more iron to body stores. The other listed options do not lead to a marked expansion of hematopoiesis.

A 3-year-old boy from Sicily has a poor appetite and is underweight for his age and height. Physical examination shows hepatosplenomegaly. The hemoglobin concentration is 6 g/dL, and the peripheral blood smear shows severely hy- pochromic and microcytic RBCs. The total serum iron level is normal, and the reticulocyte count is 10%. A radiograph of the skull shows maxillofacial deformities and expanded marrow spaces. Which of the following is the most likely cause of this child's illness? A Imbalance in α-globin and β-globin chain production B Increased fragility of erythrocyte membranes C Reduced synthesis of hemoglobin F D Relative deficiency of vitamin B12 E Sequestration of iron in reticuloendothelial cells

46 E Gram-negative sepsis with widespread endothelial damage causes disseminated intravascular coagulation. The figure shows fragmented RBCs, including "helmet cells," typical of conditions that can produce a microangiopathic hemolytic anemia, such as disseminated intravascular co- agulation, thrombocytopenia purpura, systemic lupus ery- thematosus, hemolytic-uremic syndrome, and malignant hypertension. Such fragmented RBCs are called schistocytes. In autoimmune hemolytic anemia, the hemolysis also is ex- travascular, and spherocytes are sometimes formed. Sphe- rocytes may be present in hereditary spherocytosis, but the RBC destruction is extravascular, and fragmented RBCs do not appear in the peripheral blood. Marked anisocytosis and poikilocytosis can occur in iron deficiency and in megalo- blastic anemias, but fragmentation of RBCs is not seen

A 30-year-old man has had pain and burning on urination for the past week. On physical examination, he is febrile and has a pulse of 92/min, respirations of 18/min, and blood pressure of 80/45 mm Hg. Digital rectal examination indicates that he has an enlarged, tender prostate. There is costovertebral angle ten- derness on the right. Scattered ecchymoses are present over the trunk and extremities. Laboratory studies show a blood culture positive for Klebsiella pneumoniae. The appearance of the RBCs in a peripheral blood smear is shown in the figure. Which of the following hematologic disorders is he most likely to have? A Autoimmune hemolytic anemia B Hereditary spherocytosis C Iron deficiency anemia D Megaloblastic anemia E Microangiopathic hemolytic anemia

F Glucose-6-phosphate dehydrogenase (G6PD) defici- ency predisposes the hemoglobin in RBCs to oxidative injury from drugs such as primaquine, and can induce hemolysis. Oxidant injury to hemoglobin produces inclusion of dena- tured hemoglobin within RBCs. The inclusions damage the cell membrane directly, giving rise to intravascular hemolysis. These damaged RBCs have reduced membrane deformability, and they are removed from the circulation by the spleen. The remaining mechanisms listed are not directly drug dependent. Hemolytic anemias with anti- body coating RBCs can occur with autoimmune diseases, prior transfusion, and erythroblastosis fetalis. Impaired RBC nuclear maturation occurs as a result of vitamin B12 or folate deficiency. Impaired globin synthesis occurs in thalassemias. Complement lysis is enhanced in paroxysmal nocturnal hemoglobinuria, which results from mutations in the PIGA gene. Mechanical fragmentation of RBCs is typical of microangiopathic hemolytic anemias, such as dis- seminated intravascular coagulation. Reduced RBC mem- brane deformability is seen in patients with abnormalities in cytoskeletal proteins, such as spectrin; the latter causes hereditary spherocytosis.

A 30-year-old, previously healthy man from Lagos, Nigeria, passes dark brown urine 2 days after starting the prophylactic antimalarial drug primaquine. On physical ex- amination, he appears pale and is afebrile. There is no organo- megaly. Laboratory studies show that his serum haptoglobin level is decreased. Which of the following is the most likely explanation of these findings? A Antibody-mediated hemolysis B Impaired DNA synthesis C Impaired globin chain synthesis D Increased susceptibility to complement-induced lysis E Mechanical fragmentation of RBCs as a result of vascular narrowing F Oxidative injury to hemoglobin G Reduced deformability of RBC membrane

C The infant had α-thalassemia major, which is most likely to occur in individuals of Southeast Asian ancestry, each of whose parents could have two abnormal α-globin genes on chromosome 16. A complete lack of α-globin chains precludes formation of hemoglobins A1, A2, and F. Only a tetramer of γ chains (Bart's hemoglobin) can be made, lead- ing to severe fetal anemia. Inheritance of three abnormal α-globin chains leads to hemoglobin H disease, with tetramers of β chains; survival to adulthood is possible. Hemoglobin E disease produces mild hemolytic anemias

A 32-year-old woman from Hanoi, Vietnam, gives birth at 34 weeks' gestation to a markedly hydropic stillborn male infant. Autopsy findings include hepatosplenomegaly and cardiomegaly, serous effusions in all body cavities, and gen- eralized hydrops. No congenital anomalies are noted. There is marked extramedullary hematopoiesis in visceral organs. Which of the following hemoglobins is most likely predomi- nant on hemoglobin electrophoresis of the fetal RBCs? A Hemoglobin A1 B Hemoglobin A2 C Hemoglobin Bart's D Hemoglobin E E Hemoglobin F F Hemoglobin H

C Drug-induced hemolytic anemias are neither com- mon nor severe enough to be recognized, since the hemoly- sis is mainly extravascular. However, many patients receive drugs, so the potential for a drug reaction exists, and this immune-mediated mechanism must be distinguished from other causes for anemia. Cephalosporins are the most fre- quent drugs implicated. Treatment consists of cessation of therapy with the drug, because most cases are due to drug- dependent antibody formation. Nutrient deficiencies reduce marrow production, so a reticulocytosis is unlikely. DIC is unlikely with a normal platelet count. Parvovirus infection may suppress erythropoiesis transiently in individuals with normal red cells, but may precipitate an aplastic crisis in those with a hemoglobinopathy. Persons with abnormal red cells are likely to have a history of anemia. Hemoglobinopa- thies are not Coombs positive.

A 33-year-old previously healthy man with persistent fever and heart murmur is diagnosed with infective endocar- ditis. He receives a high dosage of a cephalosporin antibiotic during the next 10 days. He now has increasing fatigue. On physical examination he has tachycardia and scleral icterus. Laboratory studies show a hemoglobin level of 7.5 g/dL, platelet count of 261,000/mm3, and total WBC count of 8300/ mm3. The direct Coombs test is positive. The periperal blood smear shows reticulocytosis. Which of the following is the most likely cause for his anemia? A Dietary nutrient deficiency B Disseminated intravascular coagulopathy C Immune-mediated hemolysis D Infection with parvovirus E Inherited hemoglobinopathy F RBC cytoskeletal protein disorder

44 A Some patients with antiphospholipid syndrome (APS) have systemic lupus erythematosus, but others (such as this patient) do not. Arterial and deep venous throm- bosis can occur in APS, with increased risk particularly of cerebral arterial thrombosis. Anticardiolipin antibody of- ten leads to a false-positive serologic test result for syphilis (Treponema pallidum infection). Thrombocytopenia is often present. APS should be considered in women who have recurrent miscarriages. Polycythemia vera is a myelopro- liferative disorder that predisposes to thrombosis, but this patient's hemoglobin value does not support this diagnosis. Thrombophlebitis occurs more frequently in pregnancy, but this explains only venous thrombosis and not the anti- cardiolipin antibodies. Trousseau syndrome, a hypercoagu- lable state associated with an underlying malignancy, can explain venous and arterial thromboses, but not the anticar- diolipin antibodies. The patient's age argues against can- cer. Von Willebrand disease is a bleeding disorder without thrombotic complications.

A 33-year-old woman, G3, P0, who has had two spon- taneous abortions, is in the second trimester of her third pregnancy. An ultrasound at 18 weeks' gestation revealed symmetric growth retardation. She gives birth to a stillborn fetus at 25 weeks, and experiences sudden onset of dyspnea. A pulmonary ventilation/perfusion scan indicates a high probability of thromboembolism. Four months later, she experiences an altered state of consciousness and sudden loss of movement in the right arm. A cerebral angiogram shows occlusion of a branch of the left middle cerebral ar- tery. Laboratory findings show hemoglobin, 13.4 g/dL; he- matocrit, 40.3%; MCV, 91 μm3; platelet count, 124,000/mm3; WBC count, 5530/mm3; prothrombin time, 13 seconds; par- tial thromboplastin time, 46 seconds; positive anticardiolip- in antibody; positive serologic test result for syphilis; and negative ANA. Which of the following best explains these findings? A Antiphospholipid syndrome B Myeloproliferative disorder C Thrombophlebitis D Treponema pallidum infection E Trousseau syndrome F Von Willebrand disease

E This patient has a warm autoimmune hemolytic anemia secondary to systemic lupus erythematosus (SLE). A positive Coombs test result indicates the presence of anti-RBC antibodies in the serum and on the RBC surface. Most cases of warm autoimmune hemolytic anemia are idiopathic, but one fourth occur in individuals with an identifiable autoimmune disease, such as SLE; in other cases, drugs are the cause. The immunoglobulin coating the RBCs acts as an opsonin to promote splenic phagocytosis. Nucleated RBCs can be seen in active hemolysis because the marrow compensates by releasing immature RBCs. Septicemia is more likely to lead to a microangiopathic hemolytic anemia. The increased RBC destruction in hereditary spherocytosis is extravascular and not immune mediated. Infections such as mononucleosis and Mycoplasma are associated with cold autoimmune hemolytic anemia (with an elevated cold agglutinin titer)

A 34-year-old woman reports becoming increasingly tired for the past 5 months. On physical examination, she is afebrile and has mild splenomegaly. Laboratory studies show a hemo- globin concentration of 10.7 g/dL and hematocrit of 32.3%. The peripheral blood smear shows spherocytes and rare nucleated RBCs. Direct and indirect Coombs test results are positive at 37° C, although not at 4° C. Which of the following underlying diseases is most likely to be diagnosed in this patient? A Escherichia coli septicemia B Hereditary spherocytosis C Infectious mononucleosis D Mycoplasma pneumoniae infection E Systemic lupus erythematosus

37 E The high MCV indicates a marked macrocytosis, greater than expected from reticulocytosis alone. The two best- known causes for such an anemia (also known as megaloblas- tic anemia when characteristic megaloblastic precursors are seen in the bone marrow) are vitamin B12 and folate deficiency. Because vitamin B12 complexed with intrinsic factor is ab- sorbed in the terminal ileum, its removal can cause vitamin B12 deficiency. Anemia of chronic disease is generally a normo- cytic anemia. Chronic blood loss and iron deficiency produce a microcytic pattern of anemia, as does dietary iron deficiency. Hemolytic anemia is unlikely several weeks after blood trans- fusion. Inflammatory bowel diseases (e.g., Crohn disease) increase the risk of malignancy, but myelophthisic anemias (from space-occupying lesions of the marrow) are usually nor- mocytic to mildly macrocytic (from reticulocytosis).

A 37-year-old woman has experienced abdominal pain and intermittent low-volume diarrhea for the past 3 months. On physical examination, she is afebrile. A stool sample is positive for occult blood. A colonoscopy is performed, and biopsy specimens from the terminal ileum and colon show mi- croscopic findings consistent with Crohn disease. She does not respond to medical therapy, and part of the colon and termi- nal ileum are removed. She is transfused with 2 U of packed RBCs during surgery. Three weeks later, she appears healthy, but complains of easy fatigability. On investigation, CBC find- ings show hemoglobin of 10.6 g/dL, hematocrit of 31.6%, RBC count of 2.69 million/μL, MCV of 118 μm3, platelet count of 378,000/mm3, and WBC count of 9800/mm3. The reticulocyte count is 0.3%. Which of the following is most likely to produce these hematologic findings? A Anemia of chronic disease B Chronic blood loss C Hemolytic anemia D Myelophthisic anemia E Vitamin B12 deficiency

47 C This patient's bleeding tendency is caused by a low platelet count. She most likely has idiopathic chronic immune thrombocytopenic purpura (ITP), in which plate- lets are destroyed in the spleen after being coated with antibodies to platelet membrane glycoproteins IIb-IIIa or Ib-IX. These antibodies coat both the patient's platelets and any transfused platelets. Because the spleen is the major source of the antibody and the site of platelet destruction, splenectomy can be beneficial if corticosteroid therapy is not. There is no defect in the production of platelets. Plate- let functions are normal in ITP. Chronic blood loss would not lead to thrombocytopenia when normal bone marrow function is present. Abnormal platelet-endothelial interac- tions are more likely to cause thrombosis. Suppression of pluripotent stem cells gives rise to aplastic anemia, which is accompanied by pancytopenia

A 37-year-old woman has noted an excessively heavy menstrual flow each of the past 6 months. She also has noticed increasing numbers of pinpoint hemorrhages on her lower ex- tremities in the past month. Physical examination shows no or- ganomegaly or lymphadenopathy. CBC shows hemoglobin of 14.2 g/dL, hematocrit of 42.5%, MCV of 91 μm3, platelet count of 15,000/mm3, and WBC count of 6950/mm3. On admission to the hospital, she has melena and after a transfusion of plate- lets, her platelet count does not increase. Which of the follow- ing describes the most likely basis for her bleeding tendency? A Abnormal production of platelets by megakaryocytes B Defective platelet-endothelial interactions C Destruction of antibody-coated platelets by the spleen D Excessive loss of platelets in menstrual blood E Suppression of pluripotent stem cell division

32 A The increased ferritin concentration and reduced total iron-binding capacity are typical of anemia of chronic disease, such as an autoimmune disease. Increased levels of cytokines such as interleukin-6 lead to increased hepatic production of hepcidin that stops ferroportin from releasing storage iron, promoting sequestration of storage iron, with poor use for erythropoiesis. Secretion of erythropoietin by the kidney is impaired. Various underlying diseases, including cancer, collagen vascular diseases, and chronic infections, can produce this pattern of anemia. Iron deficiency would pro- duce a microcytic anemia, with a low serum ferritin level and reduced hepcidin production. Aplastic anemia is unlikely because the platelet count and WBC count are normal. Mega- loblastic anemias are macrocytic without an increase in iron stores. Microangiopathic hemolytic anemias are caused by serious acute conditions such as disseminated intravascular coagulation; these patients have thrombocytopenia caused by widespread thrombosis. Thalassemia minor is uncommon and is not associated with a positive ANA test result

A 39-year-old man has experienced chronic fatigue and weight loss for the past 3 months. There are no remarkable findings on physical examination. Laboratory studies show hemoglobin, 10.0 g/dL; hematocrit, 30.3%; MCV, 91 μm3; platelet count, 240,000/mm3; WBC count, 7550/mm3; serum iron 80 μg/dL; total iron-binding capacity, 145 μg/dL; and se- rum ferritin, 565 ng/mL. Serum erythropoetin levels are low for the level of Hb and hepcidin levels are elevated. Which of the following is the most likely diagnosis? A Anemia of chronic disease B Aplastic anemia C Iron deficiency anemia D Megaloblastic anemia E Microangiopathic hemolytic anemia F Thalassemia minor

53 C Reduced numbers of platelets can result from de- creased production or increased destruction. Marrow exami- nation in this case shows numerous megakaryocytes, which excludes decreased production. Accelerated destruction can be caused by hypersplenism, but there is no splenomegaly in this case. Peripheral platelet destruction is often immunologi- cally mediated and can result from well-known autoimmune diseases such as systemic lupus erythematosus, or it can be idiopathic. When all known causes of thrombocytopenia are excluded, a diagnosis of idiopathic (immune) thrombocytope- nic purpura (ITP) can be made. This patient seems to have no other symptoms or signs and has no history of drug intake or infections that can cause thrombocytopenia. ITP is most like- ly. Thrombotic thrombocytopenic purpura (TTP) is another entity to be considered, but TTP produces a microangiopathic hemolytic anemia (MAHA) that typically is associated with fever, neurologic symptoms, and renal failure. Disseminated intravascular coagulation (DIC) is another form of MAHA. Hemophilia B, similar to hemophilia A, leads to soft tissue bleeding, and the partial thromboplastin time is prolonged, but the platelet count is normal. Metastases can act as a space- occupying lesion in the marrow to reduce hematopoiesis, but this is unlikely to be selective with megakaryocytes, and in this case, there is a megakaryocytic hyperplasia. Vitamin K deficiency prolongs the prothrombin time initially and the partial thromboplastin time if severe, but does not affect platelets. In von Willebrand disease, bleeding is due to ab- normal platelet adhesion, but platelet numbers are normal.

A 42-year-old woman has had nosebleeds, easy bruising, and increased bleeding with her menstrual periods for the past 4 months. On physical examination, her temperature is 37° C, pulse is 88/min, and blood pressure is 90/60 mm Hg. She has scattered petechiae over the distal extremities. There is no or- ganomegaly. Laboratory studies show hemoglobin of 12.3 g/ dL, hematocrit of 37%, platelet count of 21,500/mm3, and WBC count of 7370/mm3. A bone marrow biopsy specimen shows a marked increase in megakaryocytes. The prothrombin and partial thromboplastin times are within the reference range. What is the most likely diagnosis? A Disseminated intravascular coagulation B Hemophilia B C Immune thrombocytopenic purpura D Metastatic breast carcinoma E Thrombotic thrombocytopenic purpura F Vitamin K deficiency G Von Willebrand disease

40 B Her pancytopenia and absence of a reticulocytosis strongly suggest bone marrow failure. Aplastic anemia has no apparent cause in half of all cases. In other cases, drugs and tox- ins may be identified; drugs such as chemotherapeutic agents are best known for this effect. A preceding viral infection may be identified in some cases, but bacterial infections rarely cause aplastic anemias. Individuals with pancytopenia are subject to bleeding disorders because of the low platelet count and to in- fections because of the low WBC count. Dietary history would not be helpful because this patient's clinical and laboratory picture is not characteristic of iron deficiency or folate or vita- min B12 deficiency. The only known familial cause of aplastic anemia (Fanconi anemia) is rare. Menstrual history would be relevant if the patient had hypochromic microcytic anemia.

A 44-year-old woman has a 2-week history of multiple ecchymoses on her extremities after only minor trauma. She also reports feeling extremely weak. Over the previous 24 hours, she has developed a severe cough productive of yel- lowish sputum. On physical examination, her temperature is 38.4° C, and she has diffuse crackles on all lung fields. Labora- tory studies show hemoglobin, 7.2 g/dL; hematocrit, 21.4%; MCV, 88 μm3; platelet count, 35,000/mm3; and WBC count, 1400/mm3 with 20% segmented neutrophils, 1% bands, 66% lymphocytes, and 13% monocytes. The reticulocyte count is 0.1%. Which of the following historical findings would be most useful in determining the cause of her condition? A Dietary habits B Exposure to medications C Family history of anemias D Menstrual history E Recent bacterial infection

51 E This patient has hepatitis C with severe hepatocyte damage. Many of the clotting factors that are instrumental in the in vitro measurement of the extrinsic pathway of co- agulation, as measured by the prothrombin time, are synthe- sized in the liver. Increased fibrin split products suggest a consumptive coagulopathy, such as disseminated intravas- cular coagulation. Von Willebrand factor is produced by endothelial cells, not hepatocytes. Platelet aggregation is a measure of platelet function, which is not significantly affect- ed by liver disease. The platelet count is not affected directly by liver disease.

A 44-year-old woman has experienced malaise with nausea and vomiting for 3 months. On physical examination, she has scleral icterus and a yellowish hue to her skin. She has difficulty remembering three objects after 3 minutes. There are no neurologic deficits. Laboratory studies show a positive serologic test result for hepatitis C, a serum ALT of 310 U/L, AST of 269 U/L, total bilirubin of 7.6 mg/dL, direct bilirubin of 5.8 mg/dL, alkaline phosphatase of 75 U/L, and ammonia of 55 μmol/L. An abnormal result of which of the following laboratory studies of hemostatic function is most likely to be reported? A Fibrin split products B Immunoassay for plasma von Willebrand factor C Platelet aggregation D Platelet count E Prothrombin time

33 C The iron concentration and iron-binding capacity are low; however, in contrast to the finding in anemia of iron deficiency, the serum ferritin level is increased. This in- crease is typical of anemia of chronic disease. In this case, the chronic disease is rheumatoid arthritis. Underlying chronic inflammatory or neoplastic diseases increase the secretion of cytokines such as interleukin-1, tumor necrosis factor, and interferon-γ. These cytokines promote sequestration of iron in storage compartments and depress erythropoietin pro- duction. Autoantibody hemolytic anemias occur in several autoimmune diseases, such as systemic lupus erythemato- sus, but not usually in patients with rheumatoid arthritis, as in this case. Normal serum haptoglobin rules out intra- vascular hemolysis; iron is recycled at a rapid rate. Impaired synthesis of β-globin chains gives rise to β-thalassemias, also characterized by hemolysis. Complement lysis is enhanced in paroxysmal nocturnal hemoglobinuria, which results from mutations in the PIGA gene. Patients with this disorder have a history of infections. Sequestration of RBCs in the spleen occur when RBC membranes are abnormal, as in hereditary spherocytosis or sickle cell anemia, or RBCs are coated by antibodies, as in autoimmune hemolytic anemias. Metasta- ses are space-occupying lesions (myelophthisic process) that can lead to leukoerythroblastosis, with nucleated RBCs and immature WBCs appearing on the peripheral blood smear

A 45-year-old woman has experienced worsening arthritis of her hands and feet for the past 15 years. On physical examina- tion, there are marked deformities of the hands and feet, with ul- nar deviation of the hands and swan-neck deformities of the fin- gers. Laboratory studies show an elevated level of rheumatoid factor. CBC shows hemoglobin, 11.6 g/dL; hematocrit, 34.8%; MCV, 87 μm3; platelet count, 268,000/mm3; and WBC count, 6800/mm3. There is a normal serum haptoglobin level, serum iron concentration of 20 μg/dL, total iron-binding capacity of 195 μg/dL, percent saturation of 10.2, and serum ferritin concen- tration of 317 ng/mL. No fibrin split products are detected. The reticulocyte concentration is 1.1%. What is the most likely mech- anism underlying this patient's hematologic abnormalities? A Autoantibodies against RBC membranes B Impaired synthesis of β-globin chains C Inadequate usage of stored iron D Mutation in the phosphatidylinositol glycan A (PIGA) gene E Sequestration of RBCs in splenic sinusoids F Space-occupying lesions in the bone marrow

55 F The clinical features (neurologic abnormalities, fever, thrombocytopenia, microangiopathic hemolytic anemia, re- nal failure) point to thrombotic thrombocytopenic purpura (TTP), in which there is an inherited or acquired deficien- cy of the von Willebrand factor (vWF) metalloproteinase (ADAMTS13) that normally cleaves very high molecular weight multimers of vWF. The absence of ADAMTS13 gives rise to large multimers of vWF that promote widespread platelet aggregation, and the resulting microvascular occlu- sions in brain, kidney, and elsewhere produce organ dysfunc- tion, thrombocytopenia, microangiopathic hemolytic ane- mia (MAHA), and bleeding. Circulating toxins, principally endotoxins elaborated by Enterobacteriaceae such as Esch- erichia coli are important in causing endothelial injury in hemolytic-uremic syndrome (HUS). HUS has similar clinical findings to TTP, but has a different pathogenesis. Decreased factor VIII activity is a feature of hemophilia A, an X-linked disorder rare in women, characterized by bleeding into soft tissues, such as joints, and normal platelet number and func- tion. Defective aggregation of platelets in the presence of ADP and thrombin is a feature of a rare inherited disorder of platelets called Glanzmann thrombasthenia. Release of throm- boplastic substances from tumor cells or a retained dead fetus can lead to disseminated intravascular coagulation with MAHA, but this patient has no source of thromboplastins.

A 45-year-old woman has had episodes of blurred vi- sion and headaches for the past 6 months. She has had wors- ening confusion with paresthesias over the past 3 days. On physical examination, she has a temperature of 39.6° C, pulse of 100/min, respiratory rate of 20/min, and blood pressure of 80/50 mm Hg. Petechial hemorrhages are noted over her trunk and extremities. Laboratory findings include hemoglo- bin, 10.9 g/dL; hematocrit, 34%; MCV, 96 μm3; platelet count, 28,000/mm3; and WBC count, 8500/mm3. Fragmented RBCs are noted on her peripheral blood smear. Blood urea nitrogen is 40 mg/dL, and serum creatinine is 3.1 mg/dL. Which of the following is the most likely underlying cause for her findings? A Circulating toxin that injures capillary endothelium B Decreased factor VIII activity C Defective ADP-induced platelet aggregation D Formation of autoantibodies to platelet glycopro- teins IIb/IIIa and Ib-IX E Inappropriate release of thromboplastic substances into blood F Presence of antibodies against ADAMTS13 metalloproteinase

42 A She has myasthenia gravis with thymoma and red cell aplasia. The edrophonium, an acetylcholinesterase in- hibitor, will counteract the effect of the acetylcholine re- ceptor antibodies of myasthenia gravis, but not improve muscle function with antibodies against voltage-gated cal- cium channels in Lambert-Eaton myasthenic syndrome (a paraneoplastic syndrome often associated with small cell lung carcinomas). The pulmonary findings suggest heart fail- ure, and the tachycardia is consistent with high-output con- gestive heart failure from anemia. Pure red cell aplasia can be primary or arise secondarily to neoplasms, particularly thymic tumors, or autoimmune disorders. A lymphocytosis would be characteristic for lymphocytic leukemia, but this would not affect muscle strength, and lymphoblastic leukemia/lympho- ma produces mediastinal masses in much younger persons. Megakaryocytic hyperplasia would be characteristic for pe- ripheral consumption of platelets, with a disorder such as im- mune thrombocytopenic purpura (ITP), which is not likely to be associated with thymoma. Most thymomas act in a benign fashion and do not metastasize widely. Myelofibrosis could produce anemia with fatigue and weakness, but not predomi- nantly with repetitive motion. Plasmacytosis is associated with myeloma that affects bone marrow, not the mediastinum.

A 48-year-old woman has experienced increasing weak- ness and dyspnea for the past 5 months. On physical examina- tion, her temperature is 37° C, pulse 100/minute, respiratory rate 19/min, and blood pressure 115/75 mm Hg. Auscultation of the lungs reveals bilateral basilar crackles. Muscle strength diminishes from 5/5 to 4/5 with repetitive movement of her arms. Her strength returns with administration of an acetyl- cholinestrase inhibitor. A chest CT scan reveals a 6-cm cir- cumscribed anterior mediastinal mass. Which of the following findings is most likely to be present on microscopic examina- tion of her bone marrow biopsy? A Erythroid hypoplasia B Lymphocytosis C Megakaryocytic hyperplasia D Metastatic carcinoma E Myelofibrosis F Plasmacytosis

C With RBC microcytosis, iron deficiency anemia must be considered. It could be a nutritional deficiency in children and pregnant women, but more likely is due to chronic blood loss in adults. The ferritin concentration is a measure of stor- age iron because it is derived from the total body storage pool in the liver, spleen, and marrow. About 80% of func- tional body iron is contained in hemoglobin; the remainder is in muscle myoglobin. Individuals with severe liver dis- ease can have an elevated serum ferritin level because of its release from liver stores. A bone marrow biopsy specimen provides a good indication of iron stores because the iron stain of the marrow shows hemosiderin in macrophages, but such a biopsy is an expensive procedure. Some patients with hemoglobinopathies, such as β-thalassemias, also can have a microcytic anemia, but this is far less common than iron deficiency. The serum haptoglobin level is decreased with intravascular hemolysis, but the anemia is normocytic be- cause the iron can be recycled. The serum iron concentration or transferrin level by itself gives no indication of iron stores because in anemia of chronic disease, the patient's iron level can be normal to low, and the transferrin levels also can be normal to low, but iron stores are increased. Transferrin, a serum transport protein for iron, usually has about 33% iron saturation.

A 54-year-old, previously healthy man has experienced minor fatigue on exertion for the past 9 months. On physical examination, there are no remarkable findings. Laboratory studies show hemoglobin of 11.7 g/dL, hematocrit of 34.8%, MCV of 73 μm3, platelet count of 315,000/mm3, and WBC count of 8035/mm3. Which of the following is the most sensi- tive and cost-effective test that the physician should order to help to determine the cause of these findings? A Bone marrow biopsy B Hemoglobin electrophoresis C Serum ferritin D Serum haptoglobin E Serum iron F Serum transferrin

50 F The diagnosis of thrombotic thrombocytopenic pur- pura (TTP) is based on finding a classic pentad: transient neurologic problems, fever, thrombocytopenia, microangio- pathic hemolytic anemia, and acute renal failure. The diagno- sis is suggested by decreased ADAMTS13 activity. There is a deficiency of ADAMTS13, which acts as a von Willebrand factor multimer protease. Most cases of TTP are idiopathic and associated with antibodies to ADAMTS13. Hereditary TTP may result from mutations in the ADAMTS13 gene. The abnormalities are produced by small platelet-fibrin thrombi in small vessels in multiple organs. The heart, brain, and kid- ney often are severely affected. Of the other choices, only dis- seminated intravascular coagulation is a microangiopathic hemolytic anemia, but the pentad of TTP is missing

A 56-year-old woman suffers the sudden onset of head- ache and photophobia, and her condition worsens for the next 2 days. On physical examination, she has a temperature of 38° C and is disoriented. CBC shows hemoglobin of 11.2 g/dL, he- matocrit of 33.7%, MCV of 94 μm3, platelet count of 32,000/ mm3, and WBC count of 9900/mm3. The peripheral blood smear shows schistocytes. The serum urea nitrogen level is 38 mg/dL, and the creatinine level is 3.9 mg/dL. Which of the following is the most likely diagnosis? A Autoimmune hemolytic anemia B β-Thalassemia major C Disseminated intravascular coagulation D Idiopathic thrombocytopenic purpura E Paroxysmal nocturnal hemoglobinuria F Thrombotic thrombocytopenic purpura

34 A Chronic alcohol abuse can lead to folate deficiency, giving rise to megaloblastic anemia. Folic acid and vita- min B12 act as coenzymes in DNA synthetic pathways. A deficiency of either impairs the normal process of nuclear maturation. The hematopoietic cell nuclei remain large and primitive looking, giving rise to megaloblasts. The mature RBCs are larger than normal (macrocytes). Neutrophils often show defective segmentation, manifested by extra nuclear lobes. The nuclear maturation defect affects all rapidly di- viding cells in the body. Patients with chronic alcohol abuse can have thrombocytopenia and leukopenia, often because of secondary hypersplenism (alcoholic cirrhosis, leading to splenomegaly). Polychromatophilic RBCs represent reticu- locytes, and their number is reduced because of the failure of marrow to produce adequate numbers of RBCs despite anemia. Hemolytic anemias, in which antibody coats RBCs, can occur in autoimmune diseases, prior transfusion, and erythroblastosis fetalis. Hemoglobinopathies can produce a mild macrocytosis because more reticulocytes are released. An imbalance in α-globin and β-globin chain synthesis, seen in thalassemias, leads to microcytosis of RBCs. Complement lysis is enhanced in paroxysmal nocturnal hemoglobinuria, which results from mutations in the PIGA gene. Reduced RBC membrane deformability is seen in patients with abnor- malities of cytoskeletal proteins, such as spectrin; the latter causes hereditary spherocytosis

A 62-year-old man is taken to the emergency department in a state of inebriation. He is well known there because this sce- nario has been repeated many times over 15 years. On physical examination, he is afebrile. The spleen tip is palpable, and the liv- er edge is firm. Laboratory studies show hemoglobin of 8.2 g/dL, hematocrit of 25.1%, MCV of 107 μm3, platelet count of 135,000/ mm3, and WBC count of 3920/mm3. The peripheral blood smear shows prominent anisocytosis and macrocytosis. Polychromato- philic RBCs are difficult to find. A few of the neutrophils show six to seven nuclear lobes. Which of the following is the most likely explanation of these findings in his peripheral blood cells? A Diminished nuclear maturation from impaired DNA synthesis B Extravascular hemolysis of antibody-coated cells C Imbalance in synthesis of α-globin and β-globin chains D Increased susceptibility to lysis by complement E Reduced deformability of RBC membranes

D The findings point to Coombs-positive immune he- molytic anemia with indirect hyperbilirubinemia. Cold agglu- tinin immunohemolytic anemia can be seen with lymphoid neoplasms and infections such as Mycoplasma, Epstein-Barr virus, HIV, influenza virus, and cytomegalovirus (CMV). IgM binds to RBCs at cooler peripheral body regions and then fixes complement. At warmer central regions, the antibody is eluted, but the complement marks the RBCs for extravascular destruction in the spleen, but there is minimal intravascular hemolysis. The increased RBC turnover increases the MCV and the bilirubin, which is mainly indirect. Chemotherapy can suppress bone marrow production, but more likely all cell lines, and without an immune component. Although this patient has CMV infection, CMV hepatitis would likely in- crease direct and indirect bilirubin, and not account for ane- mia. Folate deficiency could account for macrocytosis, but not a positive Coombs test. Non-Hodgkin lymphomas do not of- ten involve colon, but this might account for gastrointestinal bleeding with features of iron deficiency and microcytosis.

A 65-year-old man diagnosed with follicular non-Hodgkin lymphoma is treated with chemotherapy. He develops fever and cough of a week's duration. On examination, there are bilateral pulmonary rales. A chest radiograph shows diffuse interstitial in- filtrates. A sputum specimen is positive for cytomegalovirus. He develops scleral icterus and Raynaud phenomenon. Laboratory studies show hemoglobin, 10.3 g/dL; hematocrit, 41.3%; MCV, 101 μm3; WBC count, 7600/mm3; and platelet count, 205,000/ mm3. His serum total bilirubin is 6 mg/dL, direct bilirubin is 0.8 mg/dL, and LDH is 1020 U/L. Coombs test is positive. Which of the following is the most likely mechanism for his anemia? A Marrow aplasia caused by chemotherapy B Vitamin K deficiency caused by cytomegalovirus hepatitis C Megaloblastic anemia caused by folate deficiency D Extravascular hemolysis caused by cold agglutinins E Iron deficiency caused by metastases to colon

C The marked reticulocytosis and marrow hyperpla- sia indicate that the marrow is responding to a decrease in RBCs. The reticulocytes are larger RBCs that slightly increase the MCV. An aplastic marrow is very hypocellular and un- able to respond to anemia; it is associated with pancytope- nia. The normal Coombs test results exclude an autoimmune hemolytic anemia. Iron deficiency impairs the ability of the marrow to mount a significant and sustained reticulocytosis. Iron deficiency anemia is typically microcytic and hypochro- mic, but could be partially masked here by reticulocytosis, which would not be as marked if iron were not available, but his diet is supplying needed iron. Infiltrative disorders, such as metastases in the marrow, would impair the ability to mount a reticulocytosis of this degree.

A 65-year-old man has experienced worsening fatigue for the past 5 months. On physical examination, he is afebrile and has a pulse of 91/min, respirations of 18/min, and blood pressure of 105/60 mm Hg. There is no organomegaly. A stool sample is positive for occult blood. Laboratory findings include hemoglobin of 5.9 g/dL, hematocrit of 18.3%, MCV of 99 μm3, platelet count of 250,000/mm3, and WBC count of 7800/mm3. The reticulocyte concentration is 3.9%. No fibrin split products are detected, and direct and indirect Coombs test results are negative. A bone marrow biopsy specimen shows marked erythroid hyperplasia. Which of the following conditions best explains these findings? A Aplastic anemia B Autoimmune hemolytic anemia C Chronic blood loss D Iron deficiency anemia E Metastatic carcinoma

D This child has cerebral malaria, and the smear shows numerous ring forms of the parasites in RBCs. Infection oc- curs via the bite of an Anopheles mosquito. Malarial parasites infect RBCs, causing hemolysis and anemia. Falciparum malaria is the worst form. The parasites tend to be released from cells at periodic intervals, leading to periodic fever and chills. The parasites adhere to the vascular endothelium and lead to ischemia in various organs, including the brain with consequent acute cerebral edema. There is hemolytic anemia. The liver and spleen become progressively enlarged. Babesi- osis is a rare, tick-borne disease found in the northeastern United States, which can produce a hemolytic anemia, but the organisms produce a classic "tetrad" in RBCs. Lyme dis- ease, caused by Borrelia burgdorferi, is best known for pro- ducing chronic arthritis, but meningoencephalitis, neuritis, and neuropathy may complicate this illness. Leishmaniasis, caused by Leishmania donovani, produces mainly visceral dis- ease without cerebral findings. Sleeping sickness, caused by Trypanosoma brucei, can be a chronic disease or a more acute disease causing brain dysfunction (sleeping sickness). Wuch- ereria bancrofti produces lymphatic filariasis with elephantia- sis.

A 7-year-old child has had worsening headaches and is obtunded for the past 2 days. Physical examination shows temperature of 39.5° C, pulse of 103/min, respirations of 18/ min, and blood pressure of 90/55 mm Hg. There is bilateral papilledema on funduscopic examination. No focal neuro- logic deficits are noted. Palpation of the abdomen reveals hepatosplenomegaly. Laboratory findings show hemoglobin, 9.5 g/dL; hematocrit, 28.8%; MCV, 101 μm3; platelet count, 145,000/mm3; WBC count, 6920/mm3; Na+, 146 mmol/L; K+, 5.5 mmol/L; Cl-, 106 mmol/L; CO2, 26 mmol/L; creatinine, 2.3 mg/dL; urea nitrogen, 22 mg/dL; LDH, 1095 U/L; and amylase, 45 U/L. The peripheral blood smear is shown in the figure. What infectious agent is most likely to produce these findings? A Babesia microti B Borrelia burgdorferi C Leishmania donovani D Plasmodium falciparum E Trypanosoma brucei F Wuchereria bancrofti

61 C Transfusion-related acute lung injury (TRALI) is caused when the donor plasma contains HLA or granulocytespecific antibodies which correspond to antigens found on the recipient patient WBCs. Granulocyte enzymes are released, increasing capillary permeability and resulting in sudden pulmonary edema with respiratory distress. TRALI most often occurs with administration of blood products with plasma, such as fresh frozen plasma, and more often from donor women with more HLA antibodies from prior pregnancies. TRALI may occur when the granulocytes are "primed" by sensitization within pulmonary capillaries by preexisting lung disease. Transfusion reactions to the blood proteins albumin and fibrinogen do not occur. Platelet transfusion may cause alloimmunization and subsequent platelet destruction with future platelet transfusion. Incompatible RBCs can cause alloimmunization, but more acutely, a hemolytic transfusion reaction can occur.

A 72-year-old woman undergoes laparotomy for rup- tured diverticulitis. A day later her WBC count is elevated and her blood pressure is 85/45 mm Hg. Her peripheral blood smear shows schistocytes. She receives 5 units of fresh frozen plasma. As the 5th unit is being transfused, she develops sud- den severe dyspnea and begins coughing up large quantities of frothy sputum. A chest radiograph shows bilateral pulmo- nary edema. She is most likely to have developed a transfu- sion reaction to which of the following components of the do- nor blood product? A Albumin B Fibrinogen C Granulocytes D Platelets E Red blood cells

30 E The figure shows RBC hypochromia and microcyto- sis, consistent with iron deficiency, the most common cause for anemia worldwide, either from nutritional deficiency or chronic blood loss. The lack of iron impairs heme synthesis. The marrow response is to "downsize" the RBCs, resulting in a microcytic and hypochromic anemia. At this patient's age, bleeding from an occult malignancy, such as a colonic adenocarcinoma, should be strongly suspected as the cause of iron deficiency. An autoimmune hemolytic anemia would appear as a normocytic anemia or as a slightly increased MCV with pronounced reticulocytosis. Macrocytosis would accompany a history of chronic alcoholism, probably be- cause of poor diet and folate deficiency. Thalassemias may result in a microcytosis, but β-thalassemia major causes se- vere anemia soon after birth, and survival to age 73 years is unlikely. By this patient's age, hemophilia A would have re- sulted in joint problems; because the bleeding is mainly into soft tissues without blood loss, the iron is recycled. Vitamin B12 deficiency also results in a macrocytic anemia.

A 73-year-old man takes no medications and has had no prior major illnesses or surgeries. For the past year, he has be- come increasingly tired and listless. Physical examination shows that he appears pale but has no hepatosplenomegaly and no de- formities. CBC shows hemoglobin, 9.7 g/dL; hematocrit, 29.9%; MCV, 69.7 mm3; RBC count, 4.28 million/mm3; platelet count, 331,000/mm3; and WBC count, 5500/mm3. His peripheral blood smear is shown in the figure. Which of the following is the most likely underlying condition causing this patient's findings? A Autoimmune hemolytic anemia B Chronic alcohol abuse C β-Thalassemia major D Hemophilia A E Occult malignancy F Vitamin B12 deficiency

41 E His prostatic adenocarcinoma has metastasized to the bone marrow. High alkaline phosphatase, hypercalcemia, and a leukoerythroblastic pattern in the peripheral blood (imma- ture WBCs and RBCs) are a consequence of the tumor acting as a space-occupying lesion. Such myelophthisic anemias also may be caused by infections. The anemia of chronic disease is mild. Aplastic anemias are unlikely to include leukoeryth- roblastosis. Hemolytic anemia should be accompanied by an increase in bilirubin and no abnormalities in calcium metabo- lism. The MCV in this case is not in the megaloblastic range.

A 77-year-old man has experienced increasing malaise and a 6-kg weight loss over the past year. He has noted more severe and constant back pain for the past 3 months. On physi- cal examination, his temperature is 38.7° C. His prostate is firm and irregular when palpated on digital rectal examination. There is no organomegaly. A stool sample is negative for oc- cult blood. Laboratory studies include a urine culture positive for Escherichia coli, serum glucose of 70 mg/dL, creatinine of 1.1 mg/dL, total bilirubin of 1 mg/dL, alkaline phosphatase of 293 U/L, calcium of 10.3 mg/dL, phosphorus of 2.6 mg/dL, and PSA of 25 ng/mL. CBC shows hemoglobin, 9.1 g/dL; he- matocrit, 27.3%; MCV, 94 μm3; platelet count, 55,600/mm3; and WBC count, 3570/mm3 with 18% segmented neutrophils, 7% bands, 2% metamyelocytes, 1% myelocytes, 61% lymphocytes, 11% monocytes, and 3 nucleated RBCs per 100 WBCs. What is the most likely diagnosis? A Anemia of chronic disease B Aplastic anemia C Hemolytic anemia D Megaloblastic anemia E Myelophthisic anemia

E Platelet number and function in this case are normal, and there is no detectable abnormality in the extrinsic or intrinsic pathways of coagulation as measured by the prothrombin time or partial thromboplastin time. Petechiae and ecchymoses can result from increased vascular fragility, a consequence of nutritional deficiency (e.g., vitamin C), infection (e.g., meningococcemia), and vasculitic diseases. Chronic renal failure may depress platelet function. Chronic liver disease would affect the prothrombin time. Meningococcemia is an acute illness. Metastatic disease does not directly affect hemostasis, although extensive marrow metastases could diminish platelet production.

A 77-year-old woman notices that small, pinpoint-to-blotchy areas of superficial hemorrhage have appeared on her gums and on the skin of her arms and legs over the past 3 weeks. On physical examination, she is afebrile and has no organomegaly. Laboratory studies show a normal prothrom- bin time and partial thromboplastin time. CBC shows hemo- globin of 12.7 g/dL, hematocrit of 37.2%, MCV of 80 μm3, platelet count of 276,000/mm3, and WBC count of 5600/mm3. Platelet function studies and fibrinogen level are normal, and no fibrin split products are detectable. Which of the following conditions best explains these findings? A Chronic renal failure B Macronodular cirrhosis C Meningococcemia D Metastatic carcinoma E Vitamin C deficiency

48 A Acute immune thrombocytopenic purpura (ITP) and chronic ITP are caused by antiplatelet autoantibodies, but the acute form is typically seen in children after a viral dis- ease. If the bone marrow was aplastic, all cell lines should be reduced. Glycoprotein IIb/IIIa dysfunction/deficiency can be seen with Glanzmann thrombasthenia and chronic ITP. Scurvy caused by vitamin C deficiency leads to increased capillary fragility with ecchymoses, but not to thrombocyto- penia. Von Willebrand factor metalloproteinase deficiency is a feature of thrombotic thrombocytopenic purpura.

A 9-year-old boy has developed prominent bruises on his extremities over the past week. On physical examina- tion, he has ecchymoses and petechiae on his arms and legs. Laboratory studies show hemoglobin, 13.8 g/dL; hematocrit, 41.9%; MCV, 933μm3; platelet count, 11,300/mm3; and WBC count, 7720/mm . He had respiratory syncytial virus pneumo- nia 3 weeks ago. His condition improves with corticosteroid therapy. Which of the following abnormalities is most likely to cause his hemorrhagic diathesis? A Antiplatelet antibodies B Bone marrow aplasia C Glycoprotein IIb/IIIa dysfunction D Vitamin C deficiency E Von Willebrand factor metalloproteinase deficiency

54 E A deficiency of ADAMTS13, from an acquired anti- body to this metalloproteinase or a genetic mutation in the encoding gene, can lead to accumulation of large von Wil- lebrand multimers that promote platelet microaggregate formation, resulting in TTP that is marked by a pentad of microangiopathic hemolytic anemia, fever, neurologic chang- es, thrombocytopenia, and renal failure. DIC results from ac- quired conditions that promote consumption of coagulation factors, not a metalloproteinase deficiency. HUS is very simi- lar to TTP, but is more likely related to a preceding infectious gastroenteritis with diarrhea. HIT occurs in about 5% of indi- viduals receiving heparin, and the most serious complication is widespread arterial and venous thrombosis. ITP is mainly complicated by bleeding from thrombocytopenia.

A clinical study is performed involving adult patients diagnosed with microangiopathic hemolytic anemia. A sub- group of patients who had fever or diarrhea preceding the initial diagnosis of anemia were excluded. The patients had schistocytes present on peripheral blood smears. Some of these patients were found to have a deficiency of a metalloprotein- ase known as ADAMTS13. Which of the following conditions were the patients with this deficiency most likely to have? A Disseminated intravascular coagulation (DIC) B Hemolytic-uremic syndrome (HUS) C Heparin-induced thrombocytopenia (HIT) D Idiopathic thrombocytopenic purpura (ITP) E Thrombotic thrombocytopenic purpura (TTP)

36 C Although folate and vitamin B12 deficiency both give rise to a macrocytic anemia, a deficiency of vitamin B12 also can result in demyelination of the posterior and lateral col- umns of the spinal cord. In some cases this deficiency will only be revealed by elevated levels of homocysteine and methylmalonic acid in the serum, because these are more sensitive indicators, particularly earlier in the disease. The anemia caused by vitamin B12 deficiency can be ameliorated by increased administration of folate; this masks the poten- tial neurologic injury by improving the anemia. Treating vi- tamin B12 deficiency does not improve the anemia caused by folate deficiency, however. An aplastic anemia is unlikely to result from a nutritional deficiency. Folate has no cofactor for absorption, but vitamin B12 must be complexed to intrin- sic factor and secreted by gastric parietal cells, and then the complex must be absorbed in the terminal ileum, so diseases such as atrophic gastritis and Crohn disease can affect vita- min B12 absorption more than folate. The peripheral smear could appear the same and offers no means for distinguish- ing these deficiencies.

A clinical study is performed to assess outcomes in pa- tients who have macrocytic anemias as a result of Vitamin 12 or folate deficiency. A comparison of laboratory testing strate- gies shows that the best strategy includes testing for serum homocysteine, methylmalonic acid, vitamin B12 (cobalamin), and folate. What is the most important reason for ordering these tests simultaneously? A Aplastic anemia can result from lack of either nutrient B Both nutrients are absorbed similarly C Neurologic injury must be prevented D Life-threatening thrombocytopenia can occur in both E Therapy for one deficiency also treats the other

31 C Iron absorption from the gut is tightly controlled. When body iron stores are adequate, absorption of dietary iron via DMT-1 in the duodenum is retarded, and release of iron from storage pools is inhibited. When body iron stores decrease, as with chronic blood loss, iron absorption increases. The liver-derived plasma peptide hepcidin has been found to be the iron absorption regulator. Hepcidin blocks iron absorption in the lung. Hepcidin levels increase when iron stores are high. Such fine control of iron absorption may fail, as in patients with ineffective erythropoiesis (e.g., β-thalassemia) who continue to absorb iron despite excess storage iron. Hepcidin levels are inappropriately low with both hereditary and acquired hemochromatosis. DMT-1 is an iron transporter that moves non-heme iron from the gut lumen to duodenal epithelium. Hemosiderin is an aggregated form of ferritin that does not circulate and is found only in tissues. Mutations in the HFE gene, which encodes an HLA-like transmembrane protein, lead to excessive absorption of dietary iron and hemochromatosis. Transferrin transports iron between plasma, iron stores, and developing erythroblasts.

A clinical study is performed using adult patients diagnosed with peptic ulcer disease, chronic blood loss, and hypochromic microcytic anemia. Their serum ferritin levels average 5 to 7 ng/mL. The rate of duodenal iron absorption in this study group is found to be much higher than in a nor- mal control group. After treatment with omeprazole and clar- ithromycin, study group patients have hematocrits of 40% to 42%, MCV of 82 to 85 μm3, and serum ferritin of 30 to 35 ng/ mL. Measured rates of iron absorption in the study group after therapy are now decreased to the range of the normal controls. Which of the following substances derived from liver is most likely to have been decreased in the study group patients be- fore therapy, and returned to normal after therapy? A Divalent metal transporter-1 (DMT-1) B Hemosiderin C Hepcidin D HLA-like transmembrane protein E Transferrin

C In patients with hereditary spherocytosis, spheroidal cells are trapped and destroyed in the spleen because the abnormal RBCs have reduced deformability. Splenectomy is beneficial because the spherocytes are no longer detained by the spleen. Splenectomy has no effect on the synthesis of spectrin or RBC deformability; the RBCs in spherocytosis are not killed by opsonization. In warm antibody hemolytic ane- mias, opsonized RBCs are removed by the spleen. Reactive oxygen species do not play a role in anemias. Iron is not the rate-limiting step to RBC production when the iron can be recycled within the body.

A clinical study of patients who inherit mutations that reduce the level of ankyrin, the principal binding site for spectrin, in the RBC membrane cytoskeleton shows an increased prevalence of chronic anemia with splenomegaly. For many patients, it is observed that splenectomy reduces the severity of anemia. This beneficial effect of splenectomy is most likely related to which of the following processes? A Decrease in opsonization of RBCs and lysis in spleen B Decrease in production of reactive oxygen species by splenic macrophages C Decrease in splenic RBC sequestration and lysis D Increase in deformability of RBCs within splenic sinusoids E Increase in splenic storage of iron

57 B The severity of hemophilia A depends on the amount of factor VIII activity. With less than 1% activity, there is se- vere disease, and joint hemorrhages are common, leading to severe joint deformity and ankylosis. Mild (1% to 5%) and moderate (5% to 75%) activity is often asymptomatic except in severe trauma. Hemophilia A is typically caused by decreased factor VIII activity. The affected patient is male and has male relatives who are affected (X-linked transmission). The partial thromboplastin time (PTT) is prolonged because factor VIII is required for the intrinsic pathway; the prothrombin time (PT) is normal because the extrinsic pathway does not depend on factor VIII function. The correction of the PTT by mixing with normal plasma is important. With a deficiency of factor VIII only, the addition of normal plasma, a source of factor VIII, corrects the PTT. Failure to correct PTT by normal plasma indi- cates the presence of an inhibitor in the patient's serum. About 15% of patients with hemophilia eventually develop an inhibi- tor to factor VIII. Petechiae, seen in patients with thrombocy- topenia, are not a feature of hemophilia. Factor VIII deficiency does not affect the life span of RBCs. Because individuals with factor VIII deficiency do not depend on RBC transfusions, iron overload is not a usual consequence. The bleeding tendency of hemophilia is not associated with splenomegaly.

An 11-year-old child has a history of easy bruising. At age 9 years, he experienced hemorrhaging around the phar- ynx that produced acute airway obstruction. Family history indicates that other male relatives have similar bleeding prob- lems. Laboratory studies show hemoglobin, 13.1 g/dL; hema- tocrit, 39.2%; platelet count, 228,000/mm3; WBC count, 5950/ mm3; prothrombin time, 13 seconds; and partial thromboplas- tin time, 52 seconds. A 1:1 dilution of the patient's plasma with normal pooled plasma corrects the partial thromboplastin time. Which of the following manifestations of this illness is most likely to ensue without therapy? A Conjunctival petechiae B Hemarthroses C Hemolysis D Hemochromatosis E Splenomegaly

D Patients with hereditary spherocytosis may have an aplastic crisis precipitated by a parvovirus infection. In adults who do not have a defect in normal RBC production, such as hereditary spherocytosis or sickle cell anemia, or who are not immunosuppressed, parvovirus infection is self- limited and often goes unnoticed. When there is an under- lying RBC production defect, then RBC production is shut down by parvovirus, and there is no reticulocytosis. Dissem- inated intravascular coagulation gives rise to thrombocyto- penia, bleeding, and the appearance of fragmented RBCs in the blood smear. Reticulocytosis would be prominent with hemolysis and with RBC antibodies. Iron deficiency does not occur in hemolytic anemias because the iron that is released from hemolyzed cells is reused

An 18-year-old woman from Copenhagen, Denmark, has had malaise and a low-grade fever for the past week, along with arthralgias. On physical examination, she appears very pale, except for a bright red malar facial rash. She has a history of chronic anemia, and spherocytes are observed on a peri- pheral blood smear. Her hematocrit, which normally ranges from 35% to 38%, is now 28%, and the reticulocyte count is very low. The serum bilirubin level is 0.9 mg/dL. Which of the following events is most likely to have occurred in this patient? A Accelerated extravascular hemolysis in the spleen B Development of anti-RBC antibodies C Disseminated intravascular coagulation D Reduced erythropoiesis from parvovirus infection E Superimposed dietary iron deficiency

35 B Teardrop RBCs are indicative of a myelophthisic disorder (i.e., something filling the bone marrow, such as fibrous connective tissue). The leukoerythroblastosis, in- cluding immature RBCs and WBCs, is most indicative of myelofibrosis. Splenomegaly also is typically seen in myelo- fibrosis. A leukoerythroblastic picture also can be seen in pa- tients with infections and metastases involving the marrow. Hyperplasia of erythroid normoblasts occurs in hemolytic anemias. Leukoerythroblastosis is not seen in hemolytic ane- mias. Replacement of marrow by fat occurs in aplastic ane- mia, which is characterized by pancytopenia. The presence of megaloblasts in the marrow indicates folate or vitamin B12 deficiency—both cause macrocytic anemia. Marrow packed with myeloblasts is typical of acute myeloid leukemia. In this condition, the peripheral blood also would show many myeloblasts and failure of myeloid maturation

An 83-year-old man complains of worsening malaise and fatigue over the past 5 months. On physical examination, he is afebrile and normotensive. The spleen tip is palpable. A CBC shows hemoglobin, 10.6 g/dL; hematocrit, 29.8%; MCV, 92 μm3; platelet count, 95,000/mm3; and WBC count, 4900/mm3 with 63% segmented neutrophils, 7% bands, 2% metamyelocytes, 1% myelocytes, 22% lymphocytes, 5% mono- cytes, and 3 nucleated RBCs per 100 WBCs. The peripheral blood smear shows occasional teardrop cells. An examination of the bone marrow biopsy specimen and smear is most likely to show which of the following findings? A Erythroid hyperplasia B Extensive fibrosis C Fatty replacement D Many megaloblasts E Numerous myeloblasts

A The infant most likely has erythroblastosis fetalis because maternal antibodies are coating fetal RBCs. A fetal- maternal hemorrhage in utero or at the time of delivery in a previous pregnancy (or with previous transfusion of incom- patible blood) can sensitize the mother, resulting in produc- tion of irregular IgG antibodies. In subsequent pregnancies, these antibodies (in contrast to the naturally occurring IgM antibodies) can cross the placenta to attach to fetal cells, lead- ing to hemolysis. In the past, most cases were caused by Rh incompatibility (e.g., Rh-negative mother, Rh-positive in- fant), but the use of RhoGAM administered at birth to Rh- negative mothers has eliminated almost all such cases when recognized. Other, less common blood group antigens can be involved in this process, however. The other conditions listed are not antibody mediated. A stem cell defect results in aplastic anemia and immunodeficiency. Impaired globin synthesis occurs in thalassemias. Mechanical fragmentation of RBCs is typical of microangiopathic hemolytic anemias, such as disseminated intravascular coagulation, which is more typical of pregnant women with obstetric complica- tions. Oxidative injury to hemoglobin is typical of glucose- 6-phosphate dehydrogenase (G6PD) deficiency. Reduced RBC membrane deformability is seen in patients with abnormali- ties in cytoskeletal proteins, such as spectrin; the latter causes hereditary spherocytosis

An infant is born at 34 weeks' gestation to a 28-year-old woman, G3, P2. At birth, the infant is observed to be markedly hydropic and icteric. A cord blood sample is taken, and direct Coombs test result is positive for the infant's RBCs. Which of the following is the most likely mechanism for the findings in this infant? A Hemolysis of antibody-coated cells B Hematopoietic stem cell defect C Impaired globin synthesis D Mechanical fragmentation of RBCs E Oxidative injury to hemoglobin F Reduced deformability of RBC membranes

A Haptoglobin is a serum protein that binds to free hemo- globin. Ordinarily, circulating hemoglobin is contained within RBCs, but hemolysis can release free hemoglobin. The hapto- globin is used up as the amount of free hemoglobin increases. Systemic lupus erythematosus (SLE) is an autoimmune dis- ease that can result in hemolysis by means of autoantibodies directed at RBCs, and the Coombs test result is often positive. SLE is best known to afflict young women, but it has a broad age range. Decreased iron can cause a hypochromic, micro- cytic anemia, but with hemolysis, the RBCs are recycled. He- molysis is often accompanied by reticulocytosis if the marrow is intact and the iron is not lost. An elevated D-dimer level suggests a microangiopathic hemolytic anemia. Autoimmune diseases do not affect globin chain synthesis. Protoporphyrin can be increased with some forms of porphyria.

During the past 6 months, a 25-year-old woman has noticed a malar skin rash that is made worse by sun exposure. She also has had arthralgias and myalgias. On physical exami- nation, she is afebrile and has a pulse of 100/min, respirations of 20/min, and blood pressure of 100/60 mm Hg. There is ery- thema of skin over the bridge of the nose. No organomegaly is noted. Laboratory findings include positive serologic test re- sults for ANA and double-stranded DNA, hemoglobin of 8.1 g/dL, hematocrit of 24.4%, platelet count of 87,000/mm3, and WBC count of 3950/mm3. The peripheral blood smear shows nucleated RBCs. A dipstick urinalysis is positive for blood, but there are no WBCs, RBCs, or casts seen on microscopic exami- nation of the urine. Which of the following laboratory findings is most likely to be present? A Decreased haptoglobin B Decreased iron C Decreased reticulocytosis D Elevated D dimer E Elevated hemoglobin F F Elevated protoporphyrin

39 D Telomerase is normally present in continuously dividing cells, such as hematopoietic stem cells, to prevent shortening of chromosomal telomeres; otherwise, shortened chromosomes cannot divide properly. Both genetic and ac- quired forms of aplastic anemia have been found that exhibit this mechanism. Alkaline phosphatase participates in bone remodeling. A deficiency of ADAMTS13, a metalloproteinase, can lead to accumulation of large von Willebrand multimers that promote platelet microaggregate formation, resulting in thrombotic thrombocytopenic purpura (TTP). Pyruvate kinase deficiency is one rare form of congenital anemia. Tyrosine kinases participate in cell growth regulation, and are better known to be involved with myeloproliferative disorders.

In a study of idiopathic aplastic anemia, patients are found who have premature senescence of hematopoietic stem cells. Their hematopoietic cells have normal morphology, but there are fewer cells in myeloid, erythroid, and megakaryo- cytic cell lines. Which of the following enzymes is most likely deficient in their marrow stem cells? A Alkaline phosphatase B Metalloproteinase C Pyruvate kinase D Telomerase E Tyrosine kinase

D Throughout human history, malaria has influenced the increasing gene frequency of hemoglobin S. Individu- als who are heterozygous for hemoglobin S have the sickle cell trait. They are more resistant to malaria, particularly the most malignant form caused by P. falciparum, because the parasites grow poorly or die at low oxygen concentrations, perhaps because of low potassium levels caused by potas- sium efflux from RBCs on hemoglobin sickling. The malarial parasite has difficulty completing its life cycle, even in cells with moderate amounts of hemoglobin S, as found in hetero- zygotes, giving a selective advantage to such persons living in endemic areas for falciparum malaria. Borrelia burgdorferi is the spirochete that causes Lyme disease. Clostridium per- fringens may produce gas gangrene after soft-tissue injuries. Cryptococcus neoformans can cause granulomatous disease in immunocompromised individuals. Treponema pallidum is the infectious agent causing syphilis. Trypanosoma brucei infec- tion causes sleeping sickness.

In an epidemiologic study of anemias, the findings show that there is an increased prevalence of anemia in individu- als of West African ancestry. By hemoglobin electrophoresis, some individuals within this region have increased hemoglo- bin S levels. The same regions also have a high prevalence of an infectious disease. Which of the following infectious agents is most likely to be endemic in the region where such anemia shows increased prevalence? A Borrelia burgdorferi B Clostridium perfringens C Cryptococcus neoformans D Plasmodium falciparum E Treponema pallidum F Trypanosoma brucei

C This is one mechanism for β+ thalassemia. Because the introns are usually involved, the flanking exons remain, and some normal splicing can occur, so that some β-globin chain synthesis can occur, but not sufficient for adequate he- moglobin production. The other listed mutations lead to a block in translation, with no functional β-globin chain syn- thesis, typical for β0 thalassemia.

Since childhood, a 30-year-old man has been easily fatigued with minimal exercise. Laboratory studies show hypochromic microcytic anemia. Hemoglobin electrophoresis reveals decreased Hgb A1 with increased Hgb A2 and Hgb F. His serum ferritin is markedly increased. Which of the following mutations is most likely to be present in the β-globin gene of this man? A New stop codon B Single base insertion, with frameshift C Splice site D Three-base deletion E Trinucleotide repeat

43 D Heat stroke caused by hyperthermia and loss of per- spiration from dehydration is producing hemoconcentra- tion with a relative polycythemia (note the elevated serum sodium level). Erythroleukemia is quite rare, and patients with this disorder are too ill to run a race. Chronic obstruc- tive pulmonary disease is a cause of secondary polycy- themia from chronic hypoxemia, but it does not produce hemoconcentration. Diabetes insipidus can result from a lack of antidiuretic hormone release, which leads to free water loss and dehydration, but not to hyperthermia. In- creased erythropoietin levels are seen in secondary polycy- themias, including those associated with chronic hypoxemia (high altitude or lung disease) and those associated with neoplasms secreting erythropoietin (paraneoplastic syn- drome). Polycythemia vera is a form of myeloproliferative disorder, in which erythropoietin levels are low, and there is no dehydration.

Soon after crossing the finish line in a 10-km race, a 31-year-old man collapses. On physical examination, his tem- perature is 40.1° C, pulse is 101/min, respirations are 22/min, and blood pressure is 85/50 mm Hg. He is not perspiring, and his skin shows decreased turgor. Laboratory studies show Na+, 155 mmol/L; K+, 4.6 mmol/L; Cl−, 106 mmol/L; CO2, 27 mmol/L; glucose, 68 mg/dL; creatinine, 1.8 mg/dL; hemoglo- bin, 20.1 g/dL; hematocrit, 60.3%; platelet count, 230,400/mm3; and WBC count, 6830/mm3. What is the most likely diagnosis? A Erythroleukemia B Chronic obstructive pulmonary disease C Diabetes insipidus D Hemoconcentration E Paraneoplastic syndrome F Polycythemia vera


Related study sets

Pearson VUE Exam Prep: Financing and Settlement

View Set

Nutrition Chapter 13 Water Soluble vitamins

View Set

Introduction to psychology M 32: LearningCurve 32a. Stress and Illness

View Set

XCEL Life and Health Insurance Test Review

View Set

TAMUG Moulton BIOL 112 Module 3 Final Review

View Set

algebra 2a - unit 5: rational equations lesson 19-24

View Set

Life Policy Provisions, Riders, and Options

View Set